Đến nội dung

manhtuan00 nội dung

Có 110 mục bởi manhtuan00 (Tìm giới hạn từ 14-05-2020)



Sắp theo                Sắp xếp  

#665102 Bài kiểm tra Trường Đông Toán học 2016 - Viện Toán học Hà Nội

Đã gửi bởi manhtuan00 on 19-12-2016 - 10:28 trong Thi HSG cấp Tỉnh, Thành phố. Olympic 30-4. Đề thi và kiểm tra đội tuyển các cấp.

lời giải của em cho mở rộng bài hình và bài 7 ạ
Bài 7 : 

Xét với $x = y = z = \frac{1}{3}$ thì ta có $c = \frac{9}{10}$

Ta chứng minh hằng số $c= \frac{9}{10}$ thỏa mãn đề bài

Ta có : $|x^3+y^3+z^3 -1| = |x^3+y^3+z^3 - (x+y+z)^3| = |3(x+y)(y+z)(z+x)|$

Và $|x^5+y^5+z^5-1| = |x^5+y^5+z^5-(x+y+z)^5| = |5(x+y)(y+z)(z+x)(x^2+y^2+z^2+xy+yz+zx)|$

Vậy ta cần chứng minh $|x^2+y^2+z^2+xy+yz+zx| \geq \frac{2}{3}$

Có $x^2+y^2+z^2+xy+yz+zx  = (x+y+z)^2-(xy+yz+zx) \geq \frac{2}{3}$ nên ta có điều cần chứng minh

 

Mở rộng bài 6b :

Gọi $J$ là tâm ngoại tiếp $(ADEF)$
Ta có : $\angle JAD = 90^{\circ} - \angle AED = \angle ACD$ nên $AJ$ là tiếp tuyến của $(O)$
Lấy $T$ đối xứng $A$ qua $J$. Khi đó $\angle ADT = 90^{\circ}$ nên $T,D,K$ thẳng hàng
Ta sẽ chứng minh $TH \perp AM$
Gọi $N$ đối xứng $A$ qua $M, AM$ cắt $(O)$ tại $S$. Khi đó ta có $OM \parallel KM$
nên $KN \perp BC$
$\implies KN \parallel AH \implies \angle KNS = \angle HAN = \angle DAK = \angle ATK$ nên tứ giác $TAKN$
nội tiếp . Từ đây suy ra $KN \perp NT$
Lại có tứ giác $AHNK$ là hình bình hành nên $HN \parallel AK \implies HN \perp AT$
$\implies H$ là trực tâm $\triangle ANT \implies HT \perp AM$
Vậy hình chiếu vuông góc của $H$ lên $AM$ nằm trên đường tròn $(ADEF)$

 

Hình gửi kèm

  • Untitled.png



#665131 Tuần 3 tháng 12/2016 : Đường tròn tiếp xúc

Đã gửi bởi manhtuan00 on 19-12-2016 - 12:36 trong Chuyên mục Mỗi tuần một bài toán Hình học

Lời giải của em ạ

Ta sẽ chứng minh bổ đề sau 

 

Bổ đề : $\triangle ABC$, đường tròn nội tiếp $(I)$ tiếp xúc $BC,CA,AB$ tại $D,E,F$.$AI$ cắt $DE,DF$ tại $M,N$. Gọi $G$ là trung điểm $BC$. Khi đó $G$ là tâm ngoại tiếp $\triangle DMN$

Chứng minh : $M$ là giao điểm của phân giác góc $\angle A$ và $DE$ nên $BM \perp AI$

Tương tự , $CN \perp AI$

Gọi $R$ là tiếp điểm của đường tròn bàng tiếp góc $\angle A$ với $BC$, $J$ là tâm bàng tiếp

Khi đó các tứ giác $JRMB, JNRC$ nội tiếp 

$\implies \angle DRN = \angle MJC = \angle IBD = \angle DMN$ nên tứ giác $DMRN$ nội tiếp 

Hơn nữa , $\angle DNR = \angle DNI + \angle INR = \angle DCI + \angle RCJ = 90^{\circ}$ nên $DR$ là đường kính của $(MDNR)$

$\implies G$ là tâm ngoại tiếp $(DMN)$

 

Quay lại bài toán :

Trên trung trực đoạn $A_1A_2$ lấy điểm $A_3'$ sao cho $\triangle A_2A_3'A_1 \sim \triangle BOC$

Khi đó ta có $\angle A_3'A_2A_1 = \angle OCB $ nên $OC \perp A_3'A_2$

Tương tự ta có $OB \perp A_3'A_2$

Gọi $H$ là trực tâm $\triangle ABC$

Lại có $\triangle BHC \sim \triangle A_1AA_2 \implies \triangle A_2A_3'A_1 \cap A \sim \triangle COB \cap H$

Từ đây ta có $AA_3' \perp OH$ nên $A_3' \equiv A_3$

Nên ta có $\angle A_1A_3A_2 = \angle BOC = \angle A_2A_4A_1$ . Từ đây suy ra $A_4$ là tâm ngoại tiếp $\triangle A_1AA_2$

Tương tự ta có $B_4,C_4$ là tâm ngoại tiếp $\triangle BB_1B_2, \triangle CC_1C_2$

Có $\angle AA_4A_1 = 90^{\circ} - \angle AA_2A_1 = 90^{\circ} - \angle OAC$ nên $AA_4$ là tiếp tuyến của $(O)$ tại $A$. Tương tự ta có $BB_4,CC_4 $ cũng là tiếp tuyến của $(O)$

$AA,4,BB_4,CC_4 $ cắt nhau tạo thành $\triangle UVW$

Khi đó , $(O)$ là đường tròn nội tiếp $\triangle UVW$

Áp dụng bổ đề cho $\triangle UVW$ thì $A_4,B_4,C_4$ là trung điểm $UV,VW,WU$ 

Khi đó $(A_4B_4C_4)$ là đường tròn Euler của $\triangle UVW$ và $(O)$ là đường tròn nội tiếp nên $(A_4B_4C_4)$ tiếp xúc với $(O)$ 

 

 

Hình gửi kèm

  • Untitled.png



#665149 TOPIC ôn luyện VMO 2016

Đã gửi bởi manhtuan00 on 19-12-2016 - 18:04 trong Thi HSG Quốc gia và Quốc tế

em viet nham de anh a , de dung phai la$a, b,c$ la do dai 3 canh tam giac . CMR \sum \frac{a}{b} +3 \geq 2\sum \frac{a+b}{b+c}$

Lời giải của mình, dùng S.S . Giả sử $a$ là số nhỏ nhất

Điều cần chứng minh tương đương với $\sum \frac{a}{b} - 3 \geq 2(\sum \frac{a+b}{b+c}$

Hay $(a-b)^2\frac{c^2+ca+cb-ab}{ab(a+c)(b+c)}+(a-c)(b-c)\frac{c^2+c(b-a)+ab}{ac(a+c)(a+b)}$ ( điều này đúng do $a$ là số nhỏ nhất ). Ta có điều cần chứng minh 




#665512 Chứng minh rằng: $\sum \frac{a^2}{a^2+ab+b^2}\ge \fr...

Đã gửi bởi manhtuan00 on 22-12-2016 - 20:34 trong Bất đẳng thức và cực trị

Ta có $\sum \frac{a^3}{a^2+ab+b^2} = \sum \frac{b^3}{a^2+ab+b^2}$ do $\sum \frac{a^3}{a^2+ab+b^2} - \sum \frac{b^3}{b^2+ab+a^2} = \sum a-b = 0$

Vậy ta cần chứng minh $\sum \frac{a^3+b^3}{a^2+ab+b^2} \geq \frac{2(a+b+c)}{3}$

Then BĐT Bunhiakovski ta có $(a^3+b^3)$ $\geq$ $\frac{(a^2+b^2)^2}{a+b}$ $= \frac{\frac{3a^2+3b^2}{3}.\frac{2(a^2+b^2)}{2}}{a+b} \geq \frac{\frac{2(a^2+ab+b^2)}{3}.(a+b)^2}{2(a+b)} = \frac{(a^2+ab+b^2)(a+b)}{3}$

Từ đây ta có $\sum \frac{a^3+b^3}{a^2+ab+b^2} \geq \frac{a+b}{3}$. Cộng 3 bất đẳng thức với nhau ta có điều cần chứng minh




#665527 Chứng minh rằng, với số tự nhiên $n$ bất kì đều tồn tại hai số nguy...

Đã gửi bởi manhtuan00 on 22-12-2016 - 21:00 trong Số học

Giả sử $n = \prod p_i^{\alpha_i}$ trong đó $p_i$ là các số nguyên tố

Ta chứng minh tồn tại $x,y$ để $p_i^{\alpha_i}$ $|$ $(x^2-34y^2+1)$

Thật vậy ,ta có : $x^2-34y^2+1 = (x-5y)(x+5y) -(3y-1)(3y+1)$

Nếu $p_i \neq 3$,ta chỉ cần chọn $3y \equiv 1 (\text{mod} (p_i^{\alpha_i}))$ và $x \equiv 5y (\text{mod} p_i^{\alpha_i})$ Khi đó ta có $p_i^{\alpha_i}$ $|$ $(x^2-34y^2+1)$

Nếu $p_i = 3$

Ta sẽ chứng minh điều sau bằng quy nạp : Với mọi số tự nhiên $n$, tồn tại cặp số $x,y$ để $x^2-34y^2+1$ $\vdots$ $3^k$

Với $n=1$ điều trên đúng vì ta chỉ cần chọn $x =3, y = 1$

Giả sử đúng với $n = k$, ta chứng minh đúng với $n = k+1$

Giả sử $x_0^2-34y_0^2 +1 = 3^k . m$

Nếu $m$ $\vdots$ $3$ ta có điều cần chứng minh 

Nếu $(3,m) = 1$

Dễ thấy $x_0,y_0$ không cùng chia hết cho $3$ nên tồn tại các số tự nhiên $s,t$ để $m+2sx_0-68ty_0$ $\vdots$ $3$

Khi đó, ta chọn $X = x_0+3^ks, Y = y_0+3^k.t$

$\implies X^2-34Y^2+1$ $= (x_0^2-34y_0^2+1)$ $+3^k(2s.x_0-68ty_0)+3^{2k}.s^2+34.3^{2k}t^2 = 3^k(2s.x_0-68ty_0+m)+3^{2k}.s^2+34.3^{2k}t^2$ $\vdots$ $3^{k+1}$ 

Vậy điều cần chứng minh là đúng

Tức là với mọi $p_i$ nguyên tố, tồn tại $x,y$ để $p_i^{\alpha_i}$ $|$ $(x^2-34y^2+1)$

Giả sử $p_i^{\alpha_i}$ $|$ $(x_i^2-34y_i^2+1)$

Sử dụng định lý thặng dư Trung Hoa chọn $X \equiv x_i (\text{mod} p_i)$ và $Y \equiv y_i (\text{mod} p_i) \forall i$

Khi đó $X^2 - 34Y^2+1$ $\vdots$ $n$. Ta có điều cần chứng minh




#665694 $f(f(x))=f(x)+x$

Đã gửi bởi manhtuan00 on 23-12-2016 - 23:18 trong Phương trình hàm

$f(f(x))=f(x)+x$

Từ đề bài ta có $f$ đơn ánh, mà $f$ liên tục nên $f$ đơn điệu thực sự

Thay $x$ bởi $0$ ta có $f(f(0)) = f(0)$ nên $f(0) = 0$

Ta sẽ chứng minh $f$ là song ánh

Thật vậy, giả sử tồn tại $a$ sao cho $f(x) \neq a$ $\forall x $

Do $f$ liên tục nên $f(x) > a$ hoặc $f(x) < a$ $\forall x$

Trường hợp 1 : $f(x) > a $ $\forall x$

Nếu $f$ giảm ngặt, ta có $f(x) > a \implies f(f(x)) < f(a)$

$\implies a+x < f(x) +x < f(a)$

Từ đây ta có $x < f() - a$ với mọi $x$. Ta có điều mâu thuẫn

$\implies f$ tăng ngặt. Khi đó ta có với mọi $x$ âm thì $f(x) > f(x) +x = f(f(x)) > f(a) \implies x > a $ $\forall x < 0$ ta có điều mẫu thuẫn 

Trường hợp 2 : $f(x) < a$ $\forall x$. trường hợp này có thể chứng minh tương tự trường hợp 1

Vậy $f$ song ánh nên tồn tại hàm ngược

Gọi $g(x) = f_{-1}(x)$ là hàm ngược của $f(x)$

Đặt $f_0(x) = x, f_n(x) = f(f_{n-1}(x))$, ta có dãy như sau 

$f_{n+2}(x) - f_{n+1}(x)-f_n{x} = 0$

Ta có công thức tổng quát 

$f_n(x) = \frac{f(x)-x(\frac{1-\sqrt{5}}{2})}{\sqrt{5}}.(\frac{1+\sqrt{5}}{2})^n+(\frac{x(\frac{1+\sqrt{5}}{2})-f(x)}{\sqrt{5}}).(\frac{1-\sqrt{5}}{2})^n$ $(1)$

Thay $x$ bởi $g(g(x))$ vào đẳng thức ban đầu ta được dãy : $g(g(x))+g(x) - x = 0$

$\implies g_{n+2}(x)+g_{n+1}(x) - g_n{x} = 0$

Từ đây ta thu được công thức tổng quát 

$g_n(x) = \frac{g(x)+x(\frac{1+\sqrt{5}}{2})}{\sqrt{5}}.(\frac{-1+\sqrt{5}}{2})^n+(\frac{x(\frac{-1+\sqrt{5}}{2})-g(x)}{\sqrt{5}}).(\frac{-1-\sqrt{5}}{2})^n$

Thay $x$ bởi $g(x)$ vào đẳng thức ban đầu ta có $f(x) = g(x) + x$ hay $g(x) = f(x) - x$

Vậy ta có $g_n(x) = \frac{f(x)+x(\frac{\sqrt{5}-1}{2})}{\sqrt{5}}.(\frac{-1+\sqrt{5}}{2})^n+(\frac{x(\frac{1+\sqrt{5}}{2})-f(x)}{\sqrt{5}}).(\frac{-1-\sqrt{5}}{2})^n$

Trường hợp 1 : $f$ tăng ngặt , khi đó $f_n(x), g_n(x)$ đều tăng ngặt

Ta có $f(0) = g(0) = 0 \implies f_n(0) = g_n(0) = 0$

Với mỗi $x > 0 $ thì $g_n(x) > g_n(0) = 0$ và $f_n(x) > f_n(0) = 0$

Xét $g_n(x) = \frac{f(x)+x(\frac{\sqrt{5}-1}{2})}{\sqrt{5}}.(\frac{-1+\sqrt{5}}{2})^n+(\frac{x(\frac{1+\sqrt{5}}{2})-f(x)}{\sqrt{5}}).(\frac{-1-\sqrt{5}}{2})^n$

Nếu $(\frac{x(\frac{1+\sqrt{5}}{2})-f(x)}{\sqrt{5}}) < 0$ thì ta chọn $n$ chẵn đủ lớn 

Khi $n$ đủ lớn thì $\lim_{n\rightarrow +\infty } (\frac{-1+\sqrt{5}}{2})^n = 0$ và $\lim_{n\rightarrow +\infty } (\frac{-1-\sqrt{5}}{2})^n = + \infty$ nên $g_n(x) < 0$ ta có điều mâu thuẫn

Nếu $(\frac{x(\frac{1+\sqrt{5}}{2})-f(x)}{\sqrt{5}}) > 0$ ta chọn $n$ lẻ đủ lớn , ta lại có $g_n(x) < 0$ nên mâu thuẫn

$\implies (\frac{x(\frac{1+\sqrt{5}}{2})-f(x)}{\sqrt{5}}) = 0$ từ đây ta thu được nghiệm hàm 

$f(x) = x . \frac{1+\sqrt{5}}{2}$

Trường hợp 2 : $f$ giảm ngặt . Khi đó $f_{2n}$ tăng, $f_{2n+1}$ giảm ngặt . Khi đó với mọi $x$ dương ta có $f_{2n}(x) > f_{2n}(0) = 0 = f_{2n+1}(0) > f_{2n+1}(x)$

Xét công thức tổng quát $f_n(x) = \frac{f(x)-x(\frac{1-\sqrt{5}}{2})}{\sqrt{5}}.(\frac{1+\sqrt{5}}{2})^n+(\frac{x(\frac{1+\sqrt{5}}{2})-f(x)}{\sqrt{5}}).(\frac{1-\sqrt{5}}{2})^n$ $(1)$

Làm tương tự Trường hợp 1 ta thu được nghiệm hàm $f(x) = x. \frac{1-\sqrt{5}}{2}$

Vậy các hàm thỏa mãn là $f(x) = x. \frac{1-\sqrt{5}}{2}$ và $f(x) = x . \frac{1+\sqrt{5}}{2}$ $\forall x $$\in $ $\mathbb R$

 




#665745 MAX: $P=\frac{ab}{3+c^2}+\frac{bc...

Đã gửi bởi manhtuan00 on 24-12-2016 - 18:40 trong Bất đẳng thức - Cực trị

bài này sử dụng phương pháp dồn biến 
http://thaytoan.net/...h-bat-dang-thuc

trang 16 bài số 4 




#665925 Tuần 4 tháng 12/2016 : Bài toán chia đôi cạnh

Đã gửi bởi manhtuan00 on 26-12-2016 - 20:12 trong Chuyên mục Mỗi tuần một bài toán Hình học

Cách của em hơi dài ạ 
Ta chứng minh bổ đề sau
Bổ đề : Cho $\triangle ABC$, đường tròn Mixtilinear nội tiếp xúc $(O)$ tại $D$, tiếp xúc $CA,AB$ tại $E,F$. Tiếp tuyến tại $D$ của $(O)$ cắt $BC$ tại $P$.$AD$ cắt $EF$ tại $L$. Khi đó $PL \perp AO$
Chứng minh Gọi $f(X)$ là ảnh của $X$ qua phép nghịch đảo $I^A_{AB.AC}$ hợp với phép đối xứng trục qua phân giác $\angle BAC$
Khi đó $f(B)= C, f(C) = B, f((I)) = (J)$ là đường tròn bàng tiếp $\angle A$. $f(D), f(E), f(F) $ là tiếp điểm của $(J)$ với $BC,CA,AB$. $f(L) $ là giao của $Af(D)$ với$ (Af(E)f(F))$. $f(P)$ là giao của đường tròn qua $D$ tiếp xúc $BC$ và $(ABC)$
 
Viết lại bài toán dưới cấu hình đường tròn nội tiếp, ta được bài toán tương đương : 
Cho $\triangle ABC$ với đường tròn nội tiếp $(I)$ tiếp xúc $BC,CA,AB$ tại $D,E,F$. $AH$ là đường cao ($H \in BC$). $AD$ cắt $(AEF)$ tại $M$. Đường tròn qua $A,M$ trực giao với $AH$ cắt $(O)$ tại $G$. Chứng minh rằng $(AGD)$ tiếp xúc $(O)$
 
$IM$ cắt $BC$ tại $S$. Khi đó $S,E,F$ thẳng hàng. 
Gọi $A'$ đối xứng $A$ qua $O$, $IM$ cắt $AH$ tại $R$ và $T$ là trung điểm $ID$ 
Theo 1 bài toán quen thuộc thì $R,T,A'$ thẳng hàng. Lại có $\angle AGR = 90^{\circ}$ nên $G,R,T,A'$ thẳng hàng
$X$ là trung điểm $SD$ , $AX$ cắt $(O)$ tại $G'$, $G'A'$ cắt $ID$ tại $T'$
Ta có $XD^2 = XB.XC = XG'.XA \implies (AG'D)$ tiếp xúc $(I)$ tại $D$
Gọi $K$ là tâm $(AG'D)$ .Khi đó $K,I,D$ thẳng hàng
Có $\angle XG'D = \angle XT'D = 90^{\circ} - \angle KDA \implies SH \perp AD$
Lại có $SI \perp AD \implies T'X \parallel IS \implies T'$ là trung điểm $ID$
Vậy $G \equiv G' \implies (AGD)$ tiếp xúc $BC$. Ta có điều cần chứng minh
 
Quay lại bài toán :
Tiếp tuyến của $(O)$ tại $A$ cắt $BC$ tại $S$
Ta cần chứng minh $P$ là trung điểm $QR$ . Điều cần chứng minh tương đương với $A(SP,QR) = -1$ hay $(SP,MN) = -1$
Gọi $X,Y$ lần lượt là điểm chính giữa cung nhỏ, cung lớn $BC$
$A'$ đối xứng $A$ qua $O$, $A'I$ cắt $(O)$ tại $Z$ khác $A'$. Khi đó ta có $N,D,A'$ thẳng hàng
$E,F$ là tiếp điểm của đường tròn $(K)$ với $CA,AB$ khi đó $M,I,E,F$ thẳng hàng
Ta sẽ chứng minh $M,D,X$ và $M,A,Z$ thẳng hàng 
Gọi $A_1,B_1,C_1$ là các tiếp điểm của đường tròn nội tiếp $(I)$ với $BC,CA,AB$
$\implies ZA_1$ là phân giác $\angle BZC$ nên $Z,A_1,X$ thẳng hàng
Ta có $XA_1.XZ = XI^2$ nên $\triangle XA_1I \sim \triangle XIZ$ . Từ đây ta có $\angle IMA_1 = \angle XIA_1 = \angle XZI \implies$ tứ giác $ZIA_1M$ nội tiếp. Từ đây ta có $M,Z,A$ thẳng hàng
Do $DY$ vừa là đường đối trung của $\triangle IBC$ vừa là đường phân giác của $\triangle DBC$ nên $\frac{DB}{DC}= \frac{IB^2}{IC^2}$
Ta có $\frac{MB}{MC} = \frac{ZB}{ZC}.\frac{AB}{AC}= \frac{BC_1}{CB_1}.\frac{AB}{AC}= \frac{A_1B}{A_1C}.\frac{c}{b}= \frac{p-b}{p-c} = \frac{IB^2}{IC^2}= \frac{DB}{DC}= \frac{DB}{DC}.\frac{XB}{XC} \implies M,D,X$ thẳng hàng
Theo bổ đề, ta có $DP$ là tiếp tuyến tại $D$ của $(O)$
$DS$ cắt $(O)$ tại $V$. Áp dụng định lý cho bộ 6 điểm $(AZDVXA)$
$\implies ZV$ cắt $AX$ tại $P'$ trên $BC$ . Gọi $D'$ là giao của $DP'$ với $(O)$, $ZD$ cắt $AI$ tại $W$
Có $MD.MX = MI^2 = MA_2.MP' \implies $ tứ giác $A_1P'XD$ nội tiếp $\implies ZD' \parallel BC$
$\implies (YX,ZD) = -1 = D(YX,ZD) = D(IX,WP') = (IX,WP') = Z(IX,WP') = Z(A'X,DV) = (A'X,DV) = D(A'X,DV) = D(NM,PS) = (NM,PS)$
$\implies (NM,PS) = -1$ nên $P$ là trung điểm $RQ$.

Hình gửi kèm

  • Untitled.png
  • 1232.png



#665946 Tuần 4 tháng 12/2016 : Bài toán chia đôi cạnh

Đã gửi bởi manhtuan00 on 26-12-2016 - 21:29 trong Chuyên mục Mỗi tuần một bài toán Hình học

Cách của Tuấn là theo hướng đáp án. Mình xin giới thiệu bài toán sau để mọi người cùng trao đổi.

 

Cho tam giác $ABC$ nội tiếp đường tròn $(O)$, đường tròn nội tiếp $(I)$ tiếp xúc $BC$ tại $D$. $AK$ là đường kính của $(O)$. Lấy $L$ sao cho $IL\perp AD$ và $AL\perp BC$. Chứng minh rằng $KL$ chia đôi $ID$.

 

Bài toán này có 1 phần từ AoPS và 1 phần do mình tạo ra, đây là bài toán có nội dung đẹp và chính là nguồn gốc của bài toán tuần này.

Đoạn chứng minh bổ đề em làm tắt 1 đoạn ạ, em bổ sung nốt phần chứng minh $LK$ chia đôi $ID$ ạ

$(I)$ tiếp xúc $CA,AB$ tại $E,F$.$IL$ cắt $BC$ tại $M$ khi đó $M,E,F$ thẳng hàng.

Gọi $H$ là trung điểm $MD$,$AH$ cắt $(O)$ tại $G$. $S$ là trung điểm $ID$.Theo phần trên, ta có $G,S,K$ thẳng hàng

$IL$ cắt $AD$ tại $U$. Khi đó $U,D$ đối xứng nhau qua $HS$ nên tứ giác $HUSD$ nội tiếp 

Từ đây suy ra tứ giác $HGUD$ nội tiếp. Gọi $R$ là chân đường cao hạ từ $A$ của $\triangle ABC$

Có $UL \perp AD, AL \perp BC$ nên $RLUD$ nội tiếp nên ta có $\angle ALU = \angle ADR = \angle AGU$

$\implies $ tứ giác $AGLU$ nội tiếp. Từ đây suy ra $GL \perp GA \implies G,L,S,K$ thẳng hàng




#667948 VMF's Marathon Hình học Olympic

Đã gửi bởi manhtuan00 on 10-01-2017 - 22:49 trong Hình học

$\boxed { Lời \ giải \ bài \ 109 }$ Bài toán trên tương đương với bài toán sau 

Cho $\triangle ABC$ với $I,J$ lần lượt là tâm nội tiếp và tâm bàng tiếp góc $\angle A$. $K$ là trung điểm cung lớn $BC$. $JK$ cắt $(O)$ tại $X$. $E,F$ là hình chiếu của $X$ lên $IC,IB$. Chứng minh rằng trung điểm $EF$ nằm trên trung trực $BC$

 

Chứng minh : Gọi $M,N, G$ lần lượt là tâm bàng tiếp góc $\angle B, \angle C$, trung điểm $BC$. Ta cần chứng minh $KG$ đi qua trung điểm $EF$, $JK$ cắt $BC$ tại $T$

Ta sẽ chứng minh $\frac{BF}{CE} = \frac{BM}{CN}$

Ta có : $\frac{BF}{CE} = \frac{BF}{BX} .\frac{BX}{CX} . \frac{CX}{CE} = \frac{cos \angle FBX}{cos \angle ECX} . \frac{sin \angle XCB}{sin \angle XBC} = \frac{sin \angle XBJ}{sin \angle XCJ} . \frac{sin \angle XCB}{sin \angle XBC}$

Gọi $U,V$ là trung điểm cung lớn $AC,AB$ khi đó 

$\frac{sin \angle XBJ}{sin \angle XCJ} . \frac{sin \angle XCB}{sin \angle XBC} = \frac{sin \angle XBJ}{sin \angle XCJ} . \frac{sin \angle XCB}{sin \angle XBC} $

$= \frac{sin \angle JKU}{sin \angle JKV} . \frac{sin \angle JKB}{sin \angle JKC} $
$= \frac{sin \angle KJM}{sin \angle KJN} . \frac{sin \angle JKB}{sin \angle JKC} $
$= \frac{JN}{JM} . \frac{sin \angle TKB}{sin \angle TKC} $
$= \frac{JN}{JM} . \frac{ TB}{TC} = \frac{JN}{JM} . \frac{ JM^2}{JN^2}= \frac{JM}{JN}$
Vậy ta có $\frac{BF}{CE}=  \frac{JM}{JN}$ nên theo bổ đề E.R.I.Q ta có trung điểm $BC,EF,MN$ thẳng hàng tức là trung điểm $EF$ nằm trên trung trực $BC$
 
$\boxed { Bài \ toán \ 110} $ (sách) Cho $\triangle ABC$ nội tiếp $(O)$ , $K,L$ là tâm bàng tiếp góc $\angle B, \angle C$. $(K)$ tiếp xúc $BA,BC$ tại $B_1,B_2$, tương tự ta có $C_1,C_2$. $B_1B_2$ cắt $C_1C_2$ tại $N$. Chứng minh rằng $AN \perp BC$



#667951 $f(x-f(y))=f(x)+xf(y)+f(f(y))$

Đã gửi bởi manhtuan00 on 10-01-2017 - 23:02 trong Phương trình hàm

Ta có : $f(x-f(y)) - f(x) = xf(y)+f(f(y))$. Từ đây ta có $f(x)-f(y)$ nhận mọi giá trị trên tập số thực 

$P(f(x),y) : f(f(x)-f(y)) = f(f(x))+f(x)f(y)+f(f(y)) = f(f(y)-f(x))$ từ đâ yta có $f(x) = f(-x)$ với mọi $x$

$P(0,y) : f(-f(y)) = f(0) + f(f(y))= f(f(y)) \implies f(0) = 0$

$P(f(y),y) : f(f(y)) = \frac{-f^2(y)}{2}$

Vậy ta có : $f(f(x)-f(y)) = f(x)f(y) -  \frac{-f^2(y)}{2} -  \frac{-f^2(x)}{2} =  \frac{-(f(x)-f(y))^2}{2}$ nên $f(x) = \frac{-x^2}{2}$

 




#668311 VMF's Marathon Hình học Olympic

Đã gửi bởi manhtuan00 on 14-01-2017 - 20:58 trong Hình học

Lời giải bài toán 121 : 

Gọi $J$ là giao điểm của $AP$ với $QR$. Do $AP \perp AQ$ nên $AP \parallel OR \implies R$ là trung điểm $OJ$.

$(K)$ tiếp xúc $(O)$ tại $S$. Gọi $U,V$ lần lượt là điểm chính giữa cung $AQ$ của $(O)$, cung $EF$ của $(K)$.

$SI$ là trung tuyến, $SA$ là đường đối trung $\triangle ESF$ nên $SA,SU$ đẳng giác trong $\angle ESF$. Từ đây ta có $S,U,V$ thẳng hàng

Thật vậy, theo bài toán 111 thì $DV \parallel UM$

Từ đây ta có $\triangle RUM$ và $\triangle NVD$ có các cặp cạnh tương ứng song song $\implies R,N,S$ thẳng hàng

Theo định lý Thales thì $\frac{OR}{KN} = \frac{OS}{KS} \implies OR = \frac{OS}{KS} . \frac{AK}{2}$

Ta có $\triangle QBP \sim \triangle AFK$ nên $\frac{QP}{PB} = \frac{AK}{KF} \implies \frac{2OS}{PB} = \frac{AK}{KS}$

Kết hợp 2 điều trên ta có $OR.PB = R^2 = OR.PI \implies \frac{OR}{OA} = \frac{OP}{PI} \implies \triangle RAO \sim \triangle OIP \implies \angle RQO = \angle RAO = \angle OIP$

$\implies RQ$ và $AP$ cắt nhau trên $(IOQ)$ tức là $JIOQ$ nội tiếp

Lại có $PR$ là trung tuyến $\triangle PQJ$ nên $PR$ là đường đối trung của $\triangle PIO$. Ta có điều cần chứng minh

P/s : bạn Bảo nhanh quá @@

Hình gửi kèm

  • Untitled.png



#668390 $f(x-f(y))=f(x+f(y))+4xf(y)$ $\forall x,y\in\ma...

Đã gửi bởi manhtuan00 on 15-01-2017 - 11:29 trong Phương trình hàm

Ta có $f(x-f(y)) - f(x+f(y)) = 4xf(y)$

Từ đây ta có nếu $f$ khác hằng số thì $f(x)-f(y)$ nhận mọi giá trị trên tập số thực 

Thế $x$ bởi $f(y)$ ta có $f(2f(y)) = f(0) - 4f^2(y)$

Thế $x$ bởi $2f(x) - f(y)$ ta có f(2(f(x)f(y)))=f(0)(2(f(x)f(y)))2 ; x,yR

Từ đây ta có $f(x) = f(0) - x^2$



#668408 $f(x-f(y))=f(x+f(y))+4xf(y)$ $\forall x,y\in\ma...

Đã gửi bởi manhtuan00 on 15-01-2017 - 14:00 trong Phương trình hàm

.




#668410 $f\left ( f\left ( n \right ) \right )=f\left (...

Đã gửi bởi manhtuan00 on 15-01-2017 - 14:12 trong Phương trình hàm

Bài 1 :

Giả sử tồn tại $a, m$ sao cho $f(a) = f(a+m)=k$

Khi đó ta có : $f(f(a))-f(a) = f(a+1)= f(k)  - k = f(f(a+m))-f(a+m) = f(a+m+1)$

Vậy $f(a+1) = f(a+m+1)$

Tương tự ta có $f(a+2) = f(a+m+2)$, ... $f(a+m) =f(a+2m)$

Vậy $f$ tuần hoàn chu kì $m$ nên $f$ bị chặn trên

Kí hiệu $f_k(n) = f(f(f(...(m))...)$

Ta có $f_2(n) = f(n+1)+f(n) \geq f(n)+1$

Bằng quy nạp ta có $f_k(n) \geq f_{k-1}(n)+1$

$\implies f_k(n) \geq k$

Cho $k$ tiến tới dương vô cùng ta có $f$ không bị chặn nên có điều mâu thuẫn. Vậy $f$ đơn ánh




#668503 cm với mọi $n$, luôn tồn tại số nguyên $x,y$ sao cho...

Đã gửi bởi manhtuan00 on 16-01-2017 - 01:25 trong Số học

$x^2 - 58y^2+1 = x^2 - 9y^2 - (49y^2-1) = (x-3y)(x+3y) - (7y-1)(7y+1)$

Đặt $n = \prod p_i^{\alpha_i}$

Khi $p_i \neq 7$ ta chọn $7y \equiv 1$ và $x \equiv 3y (\text{p_i^{\alpha_i}}$ thì sẽ có $p_i^{\alpha_i} | x^2 - 58y^2+1$

Khi $p_i = 7$

Ta sẽ chứng minh quy nạp : với mọi $k$ thì tồn tại $x,y$ để $7^k | x^2 - 58y^2+1$

Với $k=1$ ta chọn $x = 7, y = 2$

Giả sử giả thuyết quy nạp đúng với $k$ ta chứng minh đúng với $k+1$

Ta có $x_0^2 - 58y_0^2 +1 \vdots 7^k$. Đặt $x_0^2 - 58y_0^2 +1 = 7^k. T$. Ta có $x_0, y_0$ không cùng chia hết cho 7 nên tồn tại các số $a,b$ sao cho $a . 2x_0-b.10y_0 +T \vdots 7$

Khi đó, ta chọn $X = x_0+a.7^k, Y = y_0+b.7^k$ thì $X^2-58Y^2+1 \vdots 7^{k+1}$. Ta có điều cần chứng minh là đúng

Vậy với mỗi $p_i$ tồn tại $x_i, y_i$ để $x_i^2-58y_i^2+1 \vdots p_i^{\alpha_i}$

Ta chọn $X \equiv x_i (\text{mod p_i^{\alpha_i}}), Y  \equiv y_i (\text{mod p_i^{\alpha_i}})$ thì ta có $X^2-58Y^2+1 \vdots n$. Vậy ta có điều cần chứng minh




#668533 cm với mọi $n$, luôn tồn tại số nguyên $x,y$ sao cho...

Đã gửi bởi manhtuan00 on 16-01-2017 - 16:02 trong Số học

Vì khi $p_i \neq 7$ thì sẽ tồn tại $y$ để $7y -1 \vdots p_i^{\alpha_i}$ thì $(7y-1)(7y+1) \vdots p_i^{\alpha_i}$

Chọn tiếp $x \equiv 3y$ thì sẽ có $(x-3y)(x+3y) \vdots p_i^{\alpha_i}$ kết hợp 2 điều trên thì có $x^2-58y^2+1 \vdots p_i^{\alpha_i}$




#669145 VMF's Marathon Hình học Olympic

Đã gửi bởi manhtuan00 on 20-01-2017 - 23:22 trong Hình học

Lời giải bài 139 ( cách của em tính khá trâu bò ạ )

Ta có $(NE,AC) = -1$ nên $\frac{\overline{NA}}{\overline{NC}} = \frac{p-a}{p-c}$

Vậy tức là $\frac{\overline{NA}}{\overline{NA}+b}= \frac{p-a}{p-c}$ nên $\overline{NC} = |\frac{b(p-c)}{a-c}|$

Lại có $AQ = \frac{p-a}{cos \angle A}$ nên $CQ = b - AQ = b - \frac{p-a}{cos \angle A}$

Áp dụng đinh lý hàm cos có : $cos \angle A = \frac{b^2+c^2-a^2}{2bc}$

Vậy ta có $\overline{CQ} = \frac{b^3+bc^2-ba^2-b^2c-bc^2+abc}{b^2+c^2-a^2} = \frac{b^3-ba^2-b^2c+abc}{b^2+c^2-a^2}$

Vậy $\frac{\overline{CQ}}{\overline{CN}} = \frac{p-a}{cos \angle A}= |\frac{(b^3-ba^2-b^2c+abc)(a-c)}{(b^2+c^2-a^2)(p-c).b}| = |2\frac{(a-b)(a-c)}{b^2+c^2-a^2}|$

Tương tự ta có $\frac{\overline{BP}}{\overline{BM}} = |2\frac{(a-b)(a-c)}{b^2+c^2-a^2}|$

Nên $\frac{\overline{CQ}}{\overline{CN}}=\frac{\overline{BP}}{\overline{BM}}$ Áp dụng bổ đề E.R.I.Q  ta có trung điểm $MN,PQ,BC$ thẳng hàng

Hình gửi kèm

  • 16195453_1039342172878656_3134219046145182669_n.jpg



#669147 VMF's Marathon Hình học Olympic

Đã gửi bởi manhtuan00 on 20-01-2017 - 23:33 trong Hình học

Bài toán 140 : (sách) Cho tứ giác $ABCD$ nội tiếp. Giả sử rằng tồn tại điểm $X$ để $\angle XAD = \angle XBA = \angle XCB = \angle XDC$. Chứng minh rằng tứ giác $ABCD$ điều hòa




#669403 Bài toán T12/471 THTT

Đã gửi bởi manhtuan00 on 22-01-2017 - 19:11 trong Hình học

Lời giải của em ạ 
Gọi $X,Y$ là điểm chính giữa cung $AC,AB$ thì $Q,P$ đối xứng $A$ qua $X,Y$

Từ đây ta có $QM$ đi qua tâm bàng tiếp $I_b$ và $PN$ đi qua tâm bàng tiếp $I_c$

$I_bI_C$ cắt $BC$ tại $T$, $S$ là chân đường phân giác trong góc $\angle A$ thì $(TRMN)= (TAI_bI_c) = -1$ nên $AR$ cũng là phân giác $\angle MAN$




#669464 Tuần 4 tháng 1 năm 2017 : $JL\perp ON$

Đã gửi bởi manhtuan00 on 22-01-2017 - 22:05 trong Chuyên mục Mỗi tuần một bài toán Hình học

Gọi $W,R$ lần lượt là chân đường phân giác trong, chân đường phân giác ngoài góc $\angle A$. $T$ là chân tiếp tuyến tại $A$ nên $T$ là trung điểm $RW$.

Ta sẽ chứng minh $A,I,D,L$ đồng viên

Thật vậy, Gọi $S,Z$ là trung điểm cung lớn, cung nhỏ $BC$. 

Gọi $U$ là giao điểm của $(AEIF)$ với $(O)$. $X$ là giao của $EF$ với $BC$

Theo 1 tính chất quen thuộc thì $UD$ là phân giác $\angle BUC$ nên $U,D,Z$ thẳng hàng

$(XD,BC) = -1$ nên $UX$ là phân giác ngoài $\angle BUC$ nên $UX$ đi qua $S \implies D$ là trực tâm $\triangle SXZ$ nên $XZ \perp DS$

Từ đây ta có $D,L,S$ thẳng hàng nên $\angle LAI = \angle LSZ = \angle IDS$ nên $A,I,D,L$ đồng viên

Gọi $P'$ là tâm $ADIL$. Do $A,D,I,R$ đồng viên nên $A,D,I,R,L$ đồng viên nên $P'$ là trung điểm $IR$ 

$\implies TP' \parallel AI$ nên $p' \equiv P$ tức là $P$ là tâm $(ARLDI)$

Gọi $Z'$ đối xứng $Z$ qua $N$ thì $Z'PZM$ là hình bình hành nên $PZ' = ZM = \frac{IL}{2} \implies Z'$ là trung điểm $RL$

Gọi $V$ là trung điểm $RS$. Ta có $\angle VRZ' = \angle RIZ$ và $\angle RVZ' = \angle RSL = \angle AZL$ nên $\triangle RZ'V \sim \triangle ILZ$

Từ đây ta có $\frac{RZ'}{RV}= \frac{IL}{IZ} \implies  \frac{RZ'}{RS}= \frac{IL}{IJ}$ nên $\triangle ILJ \sim \triangle RSZ'$. Từ đây có $\angle RSZ' = \angle AJL$ mà $AJ \perp SR$ nên $LJ \perp SZ'$

$ON \parallel SZ'$ do là đường trung bình nên $ON \perp LJ$  $\square$

Hình gửi kèm

  • Untitled.png



#670522 Tuần 5 tháng 1/2017: $AR$ và trung trực $MN$ cắt nhau trê...

Đã gửi bởi manhtuan00 on 30-01-2017 - 23:51 trong Chuyên mục Mỗi tuần một bài toán Hình học

Lời giải của em ạ

Gọi $S$ là chân đường phân giác ngoài góc $\angle A$ của $\triangle ABC$. khi đó $S,P,Q$ thẳng hàng

$AR$ cắt $(I)$ tại $T$ . Ta cần chứng minh $T$ nằm trên trung trực $MN$

$AI$ cắt $(I)$ tại $G$. Ta có $\angle NAG = \angle FJG$ nên $A,N,G,D$ đồng viên, tương tự ta có $A,M,D,G$ đồng viên nên $A,M,N,D,G$ đồng viên trên $(O')$

Ta có $\angle NAG = \angle FJG = \angle EJG = \angle MAG$ nên $G$ là điểm chính giữa cung của đường tròn $(O')$

Do $JG$ là đường kính của $(I)$ nên $TJ \perp TG$ . Ta sẽ chứng minh $JT \parallel MN$ từ đó suy ra $TG \perp MN$ thì $TG$ là trung trực $MN$

Gọi $U$ là trung điểm $EF$, $V$ là giao điểm của $AS$ và $ID$

Có $A(IV,RD) = (IV,RD) = -1$ nên $AI$ là phân giác $\angle DAR$.

Gọi $H$ là giao của $DU$ với $(I)$. Khi đó ta có $\triangle FTA = \triangle EHA \sim \triangle DEA$ nên $AT,AD$ đẳng giác trong $\angle EAF$. Khi đó ta có $H \equiv T$. Vậy $T,U,D$ thẳng hàng

Ta có $IU.IA = ID^2$ nên $\angle IAT = \angle IAD = \angle UDI$ nên $A,T,I,D$ đồng viên, tức là $A,T,I,D,S$ đồng viên $\implies \angle STI = 90^{\circ} \implies ST$ là tiếp tuyến của $(I)$

Ta có $(NM,DG) = (NM,NG)+(NG,DG) = (DM,DG)+(AN,AD) = (JE,JG)+(AN,AG)+(AG,AD) = (JE,JG)+(JF,JG)+(AG,AD)= (AG,AD)$

Lại có $(JT,DG) = (JT,TS) + (TS,DS)+ (DS,DT)+(DT,DG) = (GJ,GT)+(AT,AD)+(DS,DT)+(TJ,GJ) = (AT,AD)+(DS,DT)-(TJ,TG)= (DS,DT)-(TJ,TG)+2(AG,AD) = (DS,DT)-(TJ,TG)+(DT,DI)+(AG,AD)=(DS,DI) - (TJ,TG)+(AG,AD) = (AG,AD)$

$\implies (NM,DG) = (JT,DG) \implies JT \parallel MN$. Ta có điều cần chứng minh

Hình gửi kèm

  • Untitled.png



#670524 $f(x^{2} + f(y)) = \frac{f^{2}(x)}{2} + 4y$

Đã gửi bởi manhtuan00 on 31-01-2017 - 00:21 trong Phương trình hàm

$f(x^2+f(y)) = \frac{f^2(x)}{2}+4y$

$P(0,y):f(f(y)) = \frac{f^2(0)}{2}+4y \implies f$ song ánh

$P(-x,y) \implies f^2(x) = f^2(-x) \implies f(-x) = -f(x) \implies f(0) = 0$

$P(0,y) : f(x^2) = \frac{f^2(x)}{2}$

$P(x,0) : f(f(y)) = 4y$

$\implies f(x^2+f(y))= f(x^2)+f(f(y))$

Do $f$ song ánh nên có thể thay $f(y)$ bởi $y$ nên

$f(x^2+y) = f(x^2)+f(y)$

Do $f(x^2) = \frac{f^2(x)}{2} >0$ nên $f(x^2+y) > f(y)$. Vậy $f$ tăng ngặt

Lại có $f$ cộng tính nên $f(x) = ax \implies f(x) = 2x$

 




#670608 Tuần 1 tháng 2/2017: $QR$ đi qua điểm cố định khi $P$ di...

Đã gửi bởi manhtuan00 on 07-02-2017 - 01:30 trong Chuyên mục Mỗi tuần một bài toán Hình học

Lời giải của em ạ 

Gọi $N$ đối xứng $P$ qua $O$ thì $N,E,B$ thẳng hàng và $N,F,C$ thẳng hàng. $BR,CR$ cắt $NC,NB$ tại $U,V$

$\blacksquare$ Ta chứng minh $AN,UV,BC$ đồng quy

$\angle PCR = \angle PBR$ nên $\angle VCN = \angle UBN \implies U,V,B,C$ đồng viên

$UV$ cắt $BC$ tại $T$ . Gọi $I$ là tâm $(UVBC)$. Theo định lý Brocard, $TR \perp NI$ tại $X$, $AR \perp TI$ tại $Y$

Xét nghịch đảo $ I^N_{NV.NB} : B \rightarrow V, C \rightarrow U,R \rightarrow Y, I \rightarrow X, P \rightarrow W ( W \equiv AO \cap UV)$

Thật vậy ta có $N,W,X,Y$ đồng viên trên đường tròn đường kính $NT$ nên $R,I,P$ thẳng hàng. Từ đây có $NA \perp RI$. Theo Brocard thì $NT \perp RI$ nên $N,T,A$ thẳng hàng.

$\blacksquare$ Ta chứng minh $QR$ đi qua điểm cố định 

Có $N(RT,BC) = F(ET,BC) = -1$ nên $N,Q,R$ thẳng hàng. $RQ$ cắt $(O)$ tại $L$ thì $(AL,BC) = N(AL,BC) = (TQ,BC) = -1 \implies L$ cố định. Vậy $QR$ đi qua $L$ cố định với $L$ thỏa mãn tứ giác $ABLC$ điều hòa




#670841 VMF's Marathon Bất Đẳng Thức Olympic

Đã gửi bởi manhtuan00 on 09-02-2017 - 14:37 trong Bất đẳng thức và cực trị

Bài tâp 42: Chứng mimh rằng a, b, c > 0, thì ta có 

$\frac{a}{bc}+\frac{b}{ca}+\frac{c}{ab}> \frac{2}{a} +\frac{2}{b}-\frac{2}{c}$

Điều cần chứng minh tương đương $a^2+b^2+c^2 \geq 2bc+2ca-2ab$ hay $(a+b-c)^2 \geq 0$